1. Trang chủ
  2. » Giáo án - Bài giảng

Cd2 ds10 kntt b3 phuong phap quy nap equa

12 1 0

Đang tải... (xem toàn văn)

Tài liệu hạn chế xem trước, để xem đầy đủ mời bạn chọn Tải xuống

THÔNG TIN TÀI LIỆU

Nội dung

3 PHƯƠNG PHÁP QUY NẠP TOÁN HỌC ❶ Giáo viên Soạn: Đỗ Công Lạng FB: Kẻ Ngốc Dương Trần ❷ Giáo viên phản biện :………………….… …… FB:………………………………… Kiến thức, kĩ Thuật ngữ   Mệnh đề toán học Phương pháp quy nạp tốn học    Mơ tả bước chứng minh tính đắn mệnh đề toán học phương pháp quy nạp toán học Chứng minh tính đắn mệnh đề tốn học phương pháp quy nạp toán học Vận dụng phương pháp quy nạp để giải số vấn đề thực tiễn PHƯƠNG PHÁP QUY NẠP TOÁN HỌC Trong toán học ta thường phải chứng minh mệnh đề phụ thuộc vào số tự nhiên n Phương pháp quy nạp toán học phương pháp hiệu để chứng minh mệnh đề HĐ1: Hãy quan sát đẳng thức sau: Có nhận xét số vế trái vế phải đẳng thức trên? Từ dự đốn cơng thức tính tổng n số lẻ 1+3+5+ …+(2 n−1) HĐ 2: Xét đa thức Hãy tính: ,,,, chứng tỏ kết nhận số nguyên tố Hãy đưa dự đoán cho trường hợp tổng quát Khẳng định p(n) số nguyên tố với số tự nhiên n ≥ khẳng định sai MặcChú ý: dù khẳng định với n=1,2,3,… , 40, lại sai n=41 Thật vậy, với n=41 ta có p(41)=4 12 hợp số (vì chia hết cho 41) Để khẳng định mệnh đề toán học phụ thuộc số tự nhiên n đúng, ta cần phải chứng Nhận xét: minh dù kiểm nghiệm với giá trị n Để chứng minh tính đắn mệnh đề phụ thuộc vào số tự nhiên n ∈ N ¿ , ta thử trực tiếp với số tự nhiên n ∈ N ¿ Tuy nhiên, ta tiến hành sau: Trước hết ta kiểm tra mệnh đề với n=1 Ta chứng minh rằng: từ giả thiết mệnh đề với số tự nhiên n=k ≥1, suy với n=k +1 Như mệnh đề với số tự nhiên n ∈ N ¿ Phương pháp lập luận gọi phương pháp quy nạp toán học (thường gọi tắt phương pháp quy nạp) Chứng minh mệnh đề toán học phụ thuộc n ∈ N ¿ với ∀ n ∈ N ¿, phương pháp quy nạp toán học, gồm hai bước sau: Bước Kiểm tra mệnh đề với n=1 Bước Giả thiết mệnh đề với số tự nhiên n=k ≥1( gọi giả thiết quy nạp), chứng minh mệnh đề với n=k +1 Kết luận Ví dụ Chứng minh với số tự nhiên n ≥ 1, ta có: 1+3+5+7+ …+( 2n−1)=n2 (1) Lời giải Ta chứng minh quy nạp theo n Bước Với n=1 ta có 1=12 Như (1) cho trường hợp n=1 Bước Giả sử (1) với n=kthêm điều kiện k ≥ , tức ta có 1+2+3+…+(2 k −1)=k → Giả thiết quy nạp Ta chứng minh (1) với n=k +1thêm điều kiện k ≥ 1, nghĩa ta chứng minh 1+3+5+7+ …+¿ Thật vậy, ta có: 1+3+5+7+ …+ [ 2(k + 1)−1 ] =[ 1+ 3+5+7+ …+(2 k −1) ] +(2 k +1)=k +(2k + 1) →Theo giả thiết quy nạp ¿ k +2 k +1=¿ Vậy (1) với số tự nhiên n ≥ Luyện tập Chứng minh với số tự nhiên n ≥ 1, ta có: 1+2+3+…+ n= n(n+1) ( ) ¿ Lời giải Bước 1: Với n=1 ta có: 1= 1(1+1) Vậy (¿) với n=1 Bước 2: Giả sử đẳng thức với n=k ≥1, tức ta có 1+2+3+…+ k= k (k +1) Ta chứng minh đẳng thức với n=k +1, nghĩa ta chứng minh 1+2+3+…+ k +(k +1)= (k +1)[( k +1)+1] Thật vậy, ta có: 1+2+3+…+ k +(k +1)= k (k +1) k ( k +1)+ 2(k +1) (k +1)(k +2) (k +1)[(k +1)+1] + k +1= = = 2 2 Vậy đẳng thức (¿) với số tự nhiên n ≥ Chú ý: Nếu phải chứng minh mệnh đề với số tự nhiên n ≥ p( plà số tự nhiên đó) Bước 1: Kiểm tra mệnh đề vớin=p Bước 2: Giả thiết mệnh đề với số tự nhiên n=k ≥ p chứng minh mệnh đề với n=k +1 Kết luận Ví dụ Chứng minh với số tự nhiên n ≥ 2, ta có đẳng thức: (1− 21 )(1− 31 ) …(1− n1 )= n+2 n1 (2) 2 Lời giải Ta chứng minh (2) quy nạp theo n 2+1 = = Như (2) với n=2 22 2.2 1 k +1  Giả sử (2) với n=k ,(k ≥ 2), tức là: 1− 1− … 1− = 2k k  Với n=2, ta có1− ( )( ) ( ) Ta chứng minh công thức với n=k +1, nghĩa ta chứng minh (1− 21 )(1− 31 ) …(1− k1 ) ¿ 2 Thật vậy, sử dụng giả thiết quy nạp, ta có 1 1− … 1− ¿ 2 k ( )( 1− ¿ ) ( ) k +1 (k +1−1)(k +1+1) ¿¿ 2k Vậy (2) với số tự nhiên n ≥ Luyện tập Chứng minh với số tự nhiên n ≥ 2, ta có đẳng thức a n−b n=(a−b) ( a n−1+ an−1 b +…+ ab n−2+ bn−1 ) Lời giải  Với n=2, ta có a 2−b2=(a−b)(a+ b) (đúng)  Giả sử đẳng thức với n=k ≥ nghĩa ta có: a k −b k =(a−b) ( ak−1+ ak−2 b+…+ a bk−2 +b k−1) Ta chứng minh đẳng thức với n=k +1, nghĩa ta chứng minh: a k+1−bk+1 =( a−b) ( a k + ak−1 b+…+ a bk−1 +bk ) Thật vậy, ta có a k+1−bk+1 =ak +1−a k b+ ak b−bk +1=a k (a−b)+ b(ak −b k ) ¿ a k (a−b)+ b( a−b) ( a k−1+ ak−2 b+…+ a bk−2 +b k−1 ) ¿( a−b) [ ak +b ( a k−1+ ak−2 b+…+ a bk−2 +b k−1 ) ] ¿( a−b) ( ak +a k−1 b+ …+a bk −1 +b k ) Vậy đẳng thức với số tự nhiên n ≥ 2 MỘT SỐ ỨNG DỤNG KHÁC CỦA PHƯƠNG PHÁP QUY NẠP TOÁN HỌC Trong mục ta sử dụng phương pháp quy nạp toán học để chứng minh số đẳng thức phụ thuộc số tự nhiên n Dưới ta xét số ứng dụng khác phương pháp quy nạp Ví dụ Chứng minh với số tự nhiên n, n(n+1)( n+2) chia hết cho (3) Lời giải Ta chứng minh (3) quy nạp theo n  Với n=0 ta có 0.(0+ 1)(0+2)=0 ⋮3 Vậy (3) với n=0  Giả sử(3) với n=kthêm điều kiện k ≥ 0, k ∈ N , tức k (k +1)(k +2)⋮ 3, ta cần chứng minh (3) với n=k +1 Từ giả thiết quy nạp ta suy k (k +1)(k +2)=3 m với m số tự nhiên Khi ta có (k + 1)( k+ 2)( k +3)=(k +3)(k +1)(k +2)=k (k +1)(k + 2)+3(k +1)(k +2) ¿ m+ 3(k +1)(k + 2)=3 [ m+(k +1)(k +2) ] ⋮ Vậy (3) với số tự nhiên n Vì hai số tự nhiên liên tiếp ln có số chẵn nên từ kết Ví dụ suy ra: Nhận xét: Tích ba số tự nhiên liên tiếp chia hết cho PHƯƠNG PHÁP QUY NẠP TOÁN HỌC ❶ Giáo viên Soạn: Dương Nga FB: Dương Nga ❷ Giáo viên phản biện :………………….… …… FB:………………………………… Ví dụ Chứng minh với số tự nhiên n ≥ 3, ta có 2n >2 n+1 (4 ) Lời giải Ta chứng minh bất đẳng thức ( ) quy nạp theo n, với n ≥  Với n=3 ta có 23 >7=2.3+1 Vậy ( ) với n=3  Giả sử ( ) với n=k ≥3, tức ta có 2k >2 k +1 Ta cần chứng minh ( ) với n=k +1, tức chứng minh 2k +1> ( k + )+1=2 k +3 Thật vậy, theo giả thiết quy nạp, ta có 2k +1=2 2k > ( k +1 ) =4 k +2=2 k +2 ( k +1 ) >2 k +3 k ≥ Vậy bất đẳng thức ( ) với số tự nhiên n ≥ Ví dụ 5 Sử dụng phương pháp quy nạp tốn học, chứng minh tổng góc đa giác n cạnh ( n ≥ ) ( n−2 ) 180 ° Lời giải Ta chứng minh khẳng định quy nạp theo n, với n ≥  Với n=3, ta có tổng ba góc tam giác 180 °=( 3−2 ) 180 ° Vậy khẳng định với n=3  Giả sử khẳng định với n=k ≥3, ta chứng minh với n=k +1 Thật vậy, xét đa giác k +1 cạnh A1 A Ak Ak +1, nối hai đỉnh A1 Ak ta đa giác k cạnh A1 A Ak Theo giả thiết quy nạp, tổng góc đa giác k cạnh ( k −2 ) 180 ° Dễ thấy tổng góc đa giác A1 A Ak Ak +1 tổng góc đa giác A1 A Ak cộng với tổng góc tam giác Ak +1 A k A1 , tức ( k −2 ) 180 ° +180 °=( k−1 ) 180° =[ ( k +1 )−2 ] 180 ° Vậy khẳng định với đa giác n cạnh, n ≥ Vận dụng ( Công thức lãi kép) Lãi suất gửi tiết kiệm ngân hàng thường tính theo thể thức lãi kép theo định kì Theo thể thức này, đến kì hạn người gửi khơng rút lãi tiền lãi tính vào vốn kì Giả sử người gửi số tiền A với lãi suấ r không đổi kì a) Tính tổng số tiền (cả vốn lẫn lãi) T , T , T mà người nhận sau kì thứ 1, sau kì thứ sau kì thứ b) Dự đốn cơng thức tính tổng số tiền (cả vốn lẫn lãi) T n mà người thu sau n kì Hãy chứng minh cơng thức nhận quy nạp Lời giải a) Số tiền lãi sau kỳ thứ là: Arsuy T 1= A+ A r= A (1+r ) Tương tự ta có T 2=T 1+T r = A(1+r )+ [ A(1+ r) ] r= A (1+r )(1+ r )=A ¿ T 3=T (1+ r)=A ¿ b) Dự đoán T n= A ¿ Ta chứng minh dự đoán phương pháp quy nạp  Với n=1 suy T 1= A(1+r ) (đúng)  Giả thiết công thức với n=k ≥1, ta có T k = A ¿, ta chứng minh công thức với n=k +1, nghĩa T k+1= A ¿ Ta có, cuối kỳ thứ k số tiền gốc lãi T k , sau kỳ thứ k +1 số tiền gốc lãi là: T k+1=T k +T k r=T k (1+r )= A ¿ Vậy công thức với số tự nhiên n ≥ BÀI TẬP 2.1 Sử dụng phương pháp quy nạp toán học, chứng minh đẳng thức sau với số tự nhiên n ≥ a) 2+ 4+6+ +2 n=n ( n+1 ); 2 2 b) +2 + + + n = n ( n+1 ) ( 2n+ ) Lời giải a) Chứng minh đẳng thức sau với số tự nhiên n ≥ 1: 2+ 4+6+ +2 n=n ( n+1 ) ( ) Ta chứng minh bất đẳng thức ( ) quy nạp theo n, với n ≥  Với n=1 ta có 2=1 ( 1+1 ) Vậy ( ) với n=1  Giả sử ( ) với n=k ≥1, tức ta có 2+ 4+6+ +2 k =k ( k +1 ) Ta cần chứng minh ( ) với n=k +1, tức chứng minh 2+ 4+6+ +2 k + ( k + )=( k +1 ) ( k +2 ) Thật vậy, theo giả thiết quy nạp, ta có 2+ 4+6+ +2 k + ( k + )=k ( k +1 ) +2 ( k +1 )= ( k +1 ) ( k +2 ) Vậy bất đẳng thức ( ) với số tự nhiên n ≥ 2 2 b) Chứng minh đẳng thức sau với số tự nhiên n ≥ 1: +2 + + + n = n ( n+1 ) ( 2n+ ) ( ) Ta chứng minh bất đẳng thức ( ) quy nạp theo n, với n ≥  Với n=1 ta có = ( 1+1 ) ( 2.1+1 ) Vậy ( ) với n=1  2 2 Giả sử ( ) với n=k ≥1, tức ta có +2 + + + k = k ( k +1 ) ( k +1 ) 2 2 Ta cần chứng minh ( ) với n=k +1, tức chứng minh +2 + + + k + ( k +1 ) = ( k + )( k + )( k +3 ) Thật vậy, theo giả thiết quy nạp, ta có k ( k + )( k +1 ) k ( k +1 )( k +1 ) +6 ( k +1 )2 +2 + + + k + ( k +1 ) = + ( k +1 ) = 6 2 2 ( k +1 ) [ k ( k+ )+ ( k +1 ) ] ( k +1 ) [ 2k 2+7 k +6 ] ( k +1 ) ( k +2 ) ( k +3 ) ¿ = = 6 Vậy bất đẳng thức ( ) với số tự nhiên n ≥ 2.2 Mỗi khẳng định sau hay sai? Nếu em nghĩ đúng, chứng minh Nếu em nghĩ sai, đưa phản ví dụ a) p ( n )=n2−n+11 số nguyên tố với số tự nhiên n; b) n2 >n với số tự nhiên n ≥ Lời giải a) Khẳng định “ p ( n )=n2−n+11 số nguyên tố với số tự nhiên n” khẳng định sai Phản ví dụ: lấy n=11 p ( 11 )=1 12−11+11=1 12 số nguyên tố b) Khẳng định “n2 >n ” khẳng định với số tự nhiên n ≥ Ta chứng minh bất đẳng thức n2 >n ( ¿ ) quy nạp theo n, với n ≥ Với n=2 ta có 22 >2  Vậy ( ¿ ) với n=2 Giả sử ( ¿ ) với n=k ≥ 2, tức ta có k 2> k  Ta cần chứng minh ( ¿ ) với n=k +1, tức chứng minh ( k +1 )2 > k +1 Thật vậy, theo giả thiết quy nạp, ta có ( k +1 )2 =k +2 k +1>k +2 k +1> k +1 k ≥ Vậy bất đẳng thức ( ¿ ) với số tự nhiên n ≥ 2.3 Chứng minh n3 −n+3 chia hết cho với số tự nhiên n ≥ Lời giải Ta chứng minh “n3 −n+3 chia hết cho 3” ( ¿ ) quy nạp theo n, với n ≥  Với n=1 ta có 13−1+3=3 chia hết cho Vậy ( ¿ ) với n=1  Giả sử ( ¿ ) với n=k ≥1, tức ta có “k −k +3 chia hết cho 3” Ta cần chứng minh ( ¿ ) với n=k +1, tức chứng minh “( k +1 )3 −( k +1 ) +3 chia hết cho 3” Thật vậy, theo giả thiết quy nạp suy k −k +3=3 m, với m số tự nhiên Khi ta có: ( k +1 )3 −( k +1 ) +3=k +3 k +3 k +1−k −1+ 3=( k 3−k + ) +3 ( k +k ) ¿ m+ ( k +k ) =3 ( m+k +k ) chia hết cho Vậy bất đẳng thức ( ¿ ) với số tự nhiên n ≥ 2.4 Chứng minh n2 −n+41 số lẻ với số nguyên dương n Lời giải Ta chứng minh “n2 −n+41 số lẻ ” ( ¿ ) quy nạp theo n, với n ≥ Với n=1 ta có 12−1+ 41=3 số lẻ  Vậy ( ¿ ) với n=1 Giả sử ( ¿ ) với n=k ≥1, tức ta có “k 2−k + 41 số lẻ”  Ta cần chứng minh ( ¿ ) với n=k +1, tức chứng minh “( k +1 )2 −( k +1 ) +41 số lẻ” Thật vậy, theo giả thiết quy nạp suy k 2−k + 41=2 m+1, với m số tự nhiên Khi ta có: ( k +1 )2 −( k +1 ) +41=k +2 k +1−k−1+ 41=( k 2−k + 41 ) +2 k ¿ m+1+2 k =2 ( m+ k ) +1 số lẻ Vậy bất đẳng thức ( ¿ ) với số tự nhiên n ≥ 2.5 Chứng minh x >−1 ( 1+ x )n ≥ 1+ xn với số tự nhiên n Lời giải Ta chứng minh “( 1+ x )n ≥ 1+ xn ” ( ¿ ) quy nạp theo n, với n ∈ N , x>−1  Với n=0 ta có ( 1+ x )0=1 ≥1+ x Vậy ( ¿ ) với n=0  Giả sử ( ¿ ) với n=k ≥ 0, tức ta có “( 1+ x )k ≥ 1+ x k ” Ta cần chứng minh ( ¿ ) với n=k +1, tức chứng minh “( 1+ x )k+1 ≥1+ x ( k +1 )” Thật vậy, theo giả thiết quy nạp, ta có ( 1+ x )k+1= ( 1+ x ) ( 1+ x )k ≥ ( 1+ x )( 1+kx ) =1+ kx + x+ k x2 k x ≥ ≥ 1+ kx + x=1+ x ( k+ ) Vậy bất đẳng thức ( ¿ ) với số tự nhiên 2.6 Cho tổng Sn= 1 + + + 1.2 2.3 n ( n+1 ) a) Tính S1 , S , S b) Dự đốn cơng thức tổng Sn chứng minh quy nạp Lời giải a) S1= 1 1 1 = , S2= + = , S3 = + + = 1.2 1.2 2.3 1.2 2.3 3.4 b) Dự đốn cơng thức Sn= Ta chứng minh 1 n + + + = ( ¿ ) quy nạp theo n, với n ≥ 1.2 2.3 n ( n+1 ) n+1 Với n=1 ta có  1 n + + + = 1.2 2.3 n ( n+1 ) n+ 1 = 1.2 Vậy ( ¿ ) với n=1 Giả sử ( ¿ ) với n=k ≥1, tức ta có  1 k + + + = 1.2 2.3 k ( k +1 ) k +1 Ta cần chứng minh ( ¿ ) với n=k +1, tức chứng minh 1 1 k +1 + + + + = 1.2 2.3 k ( k +1 ) ( k +1 ) ( k +2 ) k +2 Thật vậy, theo giả thiết quy nạp, ta có k ( k +2 ) +1 ( k +1 )2 1 1 k k +2 k +1 k +1 + + + + = + = = = = 1.2 2.3 k +1 k ( k +1 ) ( k +1 ) ( k +2 ) ( k+ )( k+ ) ( k+ )( k+ ) ( k +1 )( k +2 ) ( k +1 )( k +2 ) k +2 Vậy bất đẳng thức ( ¿ ) với số tự nhiên n ≥ 2.7 Sử dụng phương pháp quy nạp toán học, chứng minh số đường chéo đa giác n cạnh n ( n−3 ) ( n ≥ ) Lời giải Ta chứng minh “số đường chéo đa giác n cạnh  n ( n−3 ) ” ( ¿ ) quy nạp theo n, với n ≥ Với n=4 ta có số đường chéo tứ giác ( 4−3 ) =2 Vậy ( ¿ ) với n=4  Giả sử ( ¿ ) với n=k ≥ 4, tức ta có “số đường chéo đa giác k cạnh 10 k ( k−3 ) ” Ta cần chứng minh ( ¿ ) với n=k +1, tức chứng minh “số đường chéo đa giác k +1 cạnh ( k +1 ) ( k +1−3 ) ” Xét đa giác lồi k +1 cạnh Vẽ lại hình khác cho phù hợp với lời giải Nối A1 Ak ta đa giác k cạnh A1 A Ak , theo giả thiết quy nạp đa giác A1 A Ak có số đường k ( k−3 ) chéo Nối Ak +1 với đỉnh A1 , A , , A k−1 ta thêm k −2 đường chéo đồng thời A1 A k đường chéo Vậy số đường chéo đa giác k +1 cạnh là: k ( k−3 ) k 2−k −2 ( k+ )( k−2 ) + ( k−2 ) +1= = 2 Vậy bất đẳng thức ( ¿ ) với số tự nhiên n ≥ 2.8 Ta “lập luận” quy nạp toán học để rằng: “Mọi mèo có màu” Ta gọi P ( n ) với n nguyên dương mệnh đề sau: “Mọi mèo đàn gồm n mèo có màu” Bước Với n=1 mệnh đề P ( ) “Mọi mèo đàn gồm có màu” Hiển nhiên mệnh đề đúng! Bước Giả sử P ( k ) với số nguyên dương k Xét đàn mèo gồm k +1 Gọi chúng M , M , , M k+1 Bỏ mèo M k+1 khỏi đàn, ta nhận đàn mèo gồm k M , M , , M k Theo giả thiết quy nạp, mèo có màu Bây giờ, thay bỏ mèo M k+1 , ta bỏ mèo M để có đàn mèo gồm k M , M , , M k+1 Vẫn theo giả thiết quy nạp mèo M , M , , M k+1 có màu Cuối cùng, đưa mèo M trở lại đàn để có đàn mèo ban đầu Theo cac lập luận trên: Các mèo M , M , , M k có màu mèo M , M , , M k+1 có màu Từ suy tất mèo M , M , , M k+1 có màu Vậy, theo ngun lí quy nạp P ( n ) với số nguyên dương n Nói riêng, gọi N số mèo Trái Đất việc P ( N ) cho thấy tất mèo (trên Trái Đất) có màu! Tất nhiên ta tìm mèo khác màu nhua! Theo em “lập luận” sai chỗ nào? Lời giải 11 Lập luận sai chỗ: bỏ mèo M để có đàn mèo gồm k M , M , , M k+1 Em có biết?  Phương pháp lập luận quy nạp phát minh cá nhân thời điểm cố định Người ta cho nhà toán học Hy Lạp biết tới nguyên lí quy nạp, không thật rõ ràng  Lập luận quy nạp lần xuất cách tường minh sách Arithmeticorum Libri Duo năm 1575 nhà toán học thiên văn học người Ý Francesco Maurolico (1494 – 1575)  Nhà toán học người Anh John Wallis (1616 – 1703) coi người sử dụng thuật ngữ quy nạp 12

Ngày đăng: 17/10/2023, 05:38

TÀI LIỆU CÙNG NGƯỜI DÙNG

  • Đang cập nhật ...

TÀI LIỆU LIÊN QUAN

w